PT93.S3.Q25 - the claim that vitamin C is effective in treating acne

plong222plong222 Member
edited November 2022 in Logical Reasoning 50 karma

Can anyone explain why pt 93 s3 q25 is A and why B-E are wrong? I struggled with this one but ended up choosing D.

Comments

  • Steven_B-1Steven_B-1 Member
    795 karma

    I'll give it a whirl but weaken questions are literally my kryptonite. I ended up choosing A just because it seemed like the best answer but im always somewhat unsure on these so take my reasoning with a grain of salt.

    Okay so the argument is basically first describing an experiment. They are testing whether vitamin C is effective in treating acne. Great. The author then attacks the integrity of the study by saying "Hey! your study is flawed" because some of the subjects (half of each group) KNEW what kind of pill they were taking. So let's say we have 1000 people per group. 500 of the vitamin C group knew they were taking vitamin C and 500 given a placebo knew that it was just a placebo.

    This does seem to throw off the study b/c knowledge of treatment is a confounding factor in experiments. This then leads the author to conclude that we can tentatively conclude that vitamin C has no real beenfit in reducing the severity of acne. But then again we would still have 500 per group that do not know what treatment they received, so perhaps the study could still hold weight? That's kind of what was going through my head.

    Okay so we need to weaken and show that its not true that we can tentatively conclude that. We better fix the study or show how that flaw in the study isn't as relevant. That's how i kind of approached the question. I'm looking for an answer choice that boosts my confidence in the original study because that would then weaken the author's position.

    A - In my interpretation, this basically says hey look, even though the study may seem to have a quirk/flaw (which is debatable b/c remember that we still have 50% of participants who did not know what treatment they received), you can still trust the study because the group that took the vitamin C had severe acne and during the study they showed LESS severe acne. Hmmmm, so i took this to be kind of a counterweight to the possible objection presented against the study.
    The researches would say that we can still trust the study because there is actually evidence that vitamin C is helping people with acne since those who have historically suffered the worst cases of acne saw huge improvement and has less severe acne, even than those people who dont have a history of acne.

    B - Okay so those in that first group didn't take additional doses of vitamin C. So this would eliminate another possible confounding factor, which is that the first group saw a reduction in acne simply because they took extra doses of vitamin C while noone was looking.

    So if this didnt happen, i guess we could say that the study wasn't flawed in this respect? although the study is about vitamin C as a causal factor when treating acne, we dk if there's an upper limit on how much they're supposed to take. Plus, after a certain amount your body doesn't even use it anymore.

    Either way, this doesn't weaken the author's position at all because her objection wasn't even addressed at this point.

    C - the national average is irrelevant so i got rid of it.

    D - Okay so the first issue i saw with D was the indeterminacy of the some statement. Was it 1 or 2 of them? or 500 - 1000 of them?

    If it was just 1-2 of them, then i feel like it doesn't really do anything. Doesn't strengthen or weaken. They'll just become anomolous data points irrelevant to our general trend.

    If it was 500-1000 of them (in our imaginary 1000 person study), then if anything this answer choice strengthens the author's argument because it shows that the study is absolute poop because you weren't controlling for vitamin C intake therefore you dont have a control group lol.

    E- That's fine, those individuals were already "bad" test subjects anyway because they knew what they're treatment was going to be so what difference does it make?

    Hope this helps a bit and if someone else has a sharper analysis, please jump in. I really need to get faster at these.

    Cheers

  • 5Fennel LSAT5Fennel LSAT Member
    192 karma

    This question is challenging and potentially confusing in the setup since it introduces multiple ways of grouping the study subjects:

    Treatment vs control group - treatment is effective
    Blind treatment vs blind control - equal severity.

    The setup is important, but the point of the argument can be summarized as: If no difference in severity between blind treatment and control, then there is no real benefit (implying the observed effect is just a placebo effect because some subjects knew what they were getting a placebo). To weaken the conclusion, the answer choice must prove that there is at least some benefit to the treatment.

    The argument subtly relies on the assumption that the study is properly randomized and that the distribution initial acne severity between the treatment and control groups is uniform, which is exactly what A negates, weakening the conclusion. If A is true and the acne outcomes ended up no different, then the group that started more severe indeed gained more benefit from the treatment, so the conclusion is weakened because there is at least some benefit for the severe acne sufferers. And it is the strongest weakener as it addresses directly the relationship between both treatment and control groups.

    C does not make a distinction between the groups in the study and has no effect on the argument.

    B, D, and E are tempting as they all do address some form of distortion in the study subjects, potentially somewhat weakening the integrity of the conclusion, they fail in that they state a potential distortion affecting one group but do not exclude the other group from the same distortion, leaving the possibility open that both groups are affected by the distortion.

    Key takeaways:
    Questions about scientific studies come up frequently in LR, and the test will prey on an inclination to assume without question that the study is randomized or unbiased. One should read the stimulus critically to identify any possibilities of bias.
    Weakeners that suggest a distortion to one group without addressing the other group are incomplete and might not be the right answer.

  • AllariakAllariak Member
    51 karma

    Great explanation!

Sign In or Register to comment.